• Anúncio Global
    Respostas
    Exibições
    Última mensagem

[LIMITE] Limite com progressão...

[LIMITE] Limite com progressão...

Mensagempor mih123 » Ter Set 18, 2012 12:56

Olá, não sei resolver esse limite.Não sei nem a resposta.. ;/

\lim_{x\to\infty}\left[\frac{1}{\sqrt[2]{{n}^{2}+1}}+ \frac{1}{\sqrt[2]{{n}^{2}+2}}+\frac{1}{\sqrt[2]{{n}^{2}+3}}+...+\frac{1}{\sqrt[2]{{n}^{2}+n}}\right]}
mih123
Usuário Dedicado
Usuário Dedicado
 
Mensagens: 35
Registrado em: Seg Ago 27, 2012 03:15
Formação Escolar: GRADUAÇÃO
Andamento: cursando

Re: [LIMITE] Limite com progressão...

Mensagempor young_jedi » Ter Set 18, 2012 13:39

Só uma duvida, é limite de x tendendo ao infinito ou seria limite de n tendendo ao infinito?
young_jedi
Colaborador Voluntário
Colaborador Voluntário
 
Mensagens: 1239
Registrado em: Dom Set 09, 2012 10:48
Formação Escolar: GRADUAÇÃO
Área/Curso: Engenharia Elétrica - UEL
Andamento: formado

Re: [LIMITE] Limite com progressão...

Mensagempor mih123 » Ter Set 18, 2012 14:00

Então, na questão está x tendendo a infinito,mas eu acho que seria n tendendo a infinito.
mih123
Usuário Dedicado
Usuário Dedicado
 
Mensagens: 35
Registrado em: Seg Ago 27, 2012 03:15
Formação Escolar: GRADUAÇÃO
Andamento: cursando

Re: [LIMITE] Limite com progressão...

Mensagempor Renato_RJ » Ter Set 18, 2012 14:13

Estou um tanto confuso com essa questão... O termo n^2 não parece ser o mesmo n do último termo, veja como eu estou entendendo essa série:

\sum_{k=1}^{n} \frac{1}{\sqrt{n^2+k}}

E isso é muito estranho (ao menos para mim)....

Tem como rever a questão detalhadamente ?? Tipo, algo no enunciado por exemplo....
Iniciando a minha "caminhada" pela matemática agora... Tenho muito o quê aprender...
Avatar do usuário
Renato_RJ
Colaborador Voluntário
Colaborador Voluntário
 
Mensagens: 306
Registrado em: Qui Jan 06, 2011 15:47
Formação Escolar: PÓS-GRADUAÇÃO
Área/Curso: Mestrado em Matemática
Andamento: cursando

Re: [LIMITE] Limite com progressão...

Mensagempor young_jedi » Ter Set 18, 2012 14:15

temos que

f(n)&=&\frac{1}{\sqrt{n^2+1}}+\frac{1}{\sqrt{n^2+2}}+\frac{1}{\sqrt{n^2+3}}+...+\frac{1}{\sqrt{n^2+n}}<
\frac{1}{\sqrt{n^2}}+\frac{1}{\sqrt{n^2}}+\frac{1}{\sqrt{n^2}}+...+\frac{1}{\sqrt{n^2}}

mas

g(n)&=&\frac{1}{\sqrt{n^2}}+\frac{1}{\sqrt{n^2}}+\frac{1}{\sqrt{n^2}}+...+\frac{1}{\sqrt{n^2}}&=&\frac{n}{\sqrt{n^2}}

g(n)&=&1

temos tambem que

f(n)&=&\frac{1}{\sqrt{n^2+1}}+\frac{1}{\sqrt{n^2+2}}+\frac{1}{\sqrt{n^2+3}}+...+\frac{1}{\sqrt{n^2+n}}>
\frac{1}{\sqrt{n^2+n}}+\frac{1}{\sqrt{n^2+n}}+\frac{1}{\sqrt{n^2+n}}+...+\frac{1}{\sqrt{n^2+n}}

mas

h(n)&=&\frac{1}{\sqrt{n^2+n}}+\frac{1}{\sqrt{n^2+n}}+\frac{1}{\sqrt{n^2+n}}+...+\frac{1}{\sqrt{n^2+n}}&=&\frac{n}{\sqrt{n^2+n}}

mas

\lim_{n\rightarrow\infty}\frac{n}{\sqrt{n^2+n}}&=&\lim_{n\rightarrow\infty}\frac{1}{\sqrt{1+\frac{1}{n}}}

\lim_{n\rightarrow\infty}\frac{1}{\sqrt{1+\frac{1}{n}}}&=&1

pelo teorema do confronto se

h(x)<f(x)<g(x)

e

\lim_{x\rightarrow\infty}h(x)&=&a}

\lim_{x\rightarrow\infty}g(x)&=&a}

então

\lim_{x\rightarrow\infty}f(x)&=&a}

com isso temos

\lim_{n\rightarrow\infty}f(n)&=&1}
young_jedi
Colaborador Voluntário
Colaborador Voluntário
 
Mensagens: 1239
Registrado em: Dom Set 09, 2012 10:48
Formação Escolar: GRADUAÇÃO
Área/Curso: Engenharia Elétrica - UEL
Andamento: formado

Re: [LIMITE] Limite com progressão...

Mensagempor mih123 » Ter Set 18, 2012 14:36

No enunciado diz apenas pra determinar o limite.Quando tiver aula novamente com o professor eu pergunto.

Valew :-D
mih123
Usuário Dedicado
Usuário Dedicado
 
Mensagens: 35
Registrado em: Seg Ago 27, 2012 03:15
Formação Escolar: GRADUAÇÃO
Andamento: cursando

Re: [LIMITE] Limite com progressão...

Mensagempor Renato_RJ » Ter Set 18, 2012 14:53

mih123 escreveu:No enunciado diz apenas pra determinar o limite.Quando tiver aula novamente com o professor eu pergunto.

Valew :-D


Opa, o colega Young_Jedi matou a questão !!
Iniciando a minha "caminhada" pela matemática agora... Tenho muito o quê aprender...
Avatar do usuário
Renato_RJ
Colaborador Voluntário
Colaborador Voluntário
 
Mensagens: 306
Registrado em: Qui Jan 06, 2011 15:47
Formação Escolar: PÓS-GRADUAÇÃO
Área/Curso: Mestrado em Matemática
Andamento: cursando


Re: [LIMITE] Limite com progressão...

Mensagempor young_jedi » Ter Set 18, 2012 15:15

porque ai voce so colocou quatro termos da serie
mais a quantidades de termos depende de n tambem
young_jedi
Colaborador Voluntário
Colaborador Voluntário
 
Mensagens: 1239
Registrado em: Dom Set 09, 2012 10:48
Formação Escolar: GRADUAÇÃO
Área/Curso: Engenharia Elétrica - UEL
Andamento: formado

Re: [LIMITE] Limite com progressão...

Mensagempor mih123 » Qua Set 19, 2012 02:17

young_jedi escreveu:temos que

f(n)&=&\frac{1}{\sqrt{n^2+1}}+\frac{1}{\sqrt{n^2+2}}+\frac{1}{\sqrt{n^2+3}}+...+\frac{1}{\sqrt{n^2+n}}<
\frac{1}{\sqrt{n^2}}+\frac{1}{\sqrt{n^2}}+\frac{1}{\sqrt{n^2}}+...+\frac{1}{\sqrt{n^2}}

mas

g(n)&=&\frac{1}{\sqrt{n^2}}+\frac{1}{\sqrt{n^2}}+\frac{1}{\sqrt{n^2}}+...+\frac{1}{\sqrt{n^2}}&=&\frac{n}{\sqrt{n^2}}

g(n)&=&1

temos tambem que

f(n)&=&\frac{1}{\sqrt{n^2+1}}+\frac{1}{\sqrt{n^2+2}}+\frac{1}{\sqrt{n^2+3}}+...+\frac{1}{\sqrt{n^2+n}}>
\frac{1}{\sqrt{n^2+n}}+\frac{1}{\sqrt{n^2+n}}+\frac{1}{\sqrt{n^2+n}}+...+\frac{1}{\sqrt{n^2+n}}

mas

h(n)&=&\frac{1}{\sqrt{n^2+n}}+\frac{1}{\sqrt{n^2+n}}+\frac{1}{\sqrt{n^2+n}}+...+\frac{1}{\sqrt{n^2+n}}&=&\frac{n}{\sqrt{n^2+n}}

mas

\lim_{n\rightarrow\infty}\frac{n}{\sqrt{n^2+n}}&=&\lim_{n\rightarrow\infty}\frac{1}{\sqrt{1+\frac{1}{n}}}

\lim_{n\rightarrow\infty}\frac{1}{\sqrt{1+\frac{1}{n}}}&=&1

pelo teorema do confronto se

h(x)<f(x)<g(x)

e

\lim_{x\rightarrow\infty}h(x)&=&a}

\lim_{x\rightarrow\infty}g(x)&=&a}

então

\lim_{x\rightarrow\infty}f(x)&=&a}

com isso temos

\lim_{n\rightarrow\infty}f(n)&=&1}


Boa Noite, Young_jedi estou muito confusa com sua resolução, não consegui entender porque ficou \frac{1}{\sqrt[2]{{n}^{2}}} e na outra função \frac{1} {\sqrt[2]{{n}^{2}+n}} ??
mih123
Usuário Dedicado
Usuário Dedicado
 
Mensagens: 35
Registrado em: Seg Ago 27, 2012 03:15
Formação Escolar: GRADUAÇÃO
Andamento: cursando

Re: [LIMITE] Limite com progressão...

Mensagempor MarceloFantini » Qua Set 19, 2012 02:27

Ele não falou que é igual, e sim que é menor. Note que n^2 + n > n^2 para todo valor natural, então \sqrt{n^2 +n} > \sqrt{n^2} e \frac{1}{\sqrt{n^2 +n}} < \frac{1}{\sqrt{n^2}}.
Futuro MATEMÁTICO
e^{\pi \cdot i} +1 = 0
MarceloFantini
Colaborador Moderador
Colaborador Moderador
 
Mensagens: 3126
Registrado em: Seg Dez 14, 2009 11:41
Formação Escolar: GRADUAÇÃO
Andamento: formado

Re: [LIMITE] Limite com progressão...

Mensagempor mih123 » Qua Set 19, 2012 13:55

Ahh sim. Olhei direitinho, entendi! Muito Obrigada.
mih123
Usuário Dedicado
Usuário Dedicado
 
Mensagens: 35
Registrado em: Seg Ago 27, 2012 03:15
Formação Escolar: GRADUAÇÃO
Andamento: cursando


Voltar para Cálculo: Limites, Derivadas e Integrais

 



  • Tópicos relacionados
    Respostas
    Exibições
    Última mensagem

Quem está online

Usuários navegando neste fórum: Nenhum usuário registrado e 78 visitantes

 



Assunto: Unesp - 95 Números Complexos
Autor: Alucard014 - Dom Ago 01, 2010 18:22

(UNESP - 95) Seja L o Afixo de um Número complexo a=\sqrt{8}+ i em um sistema de coordenadas cartesianas xOy. Determine o número complexo b , de módulo igual a 1 , cujo afixo M pertence ao quarto quadrante e é tal que o ângulo LÔM é reto.


Assunto: Unesp - 95 Números Complexos
Autor: MarceloFantini - Qui Ago 05, 2010 17:27

Seja \alpha o ângulo entre o eixo horizontal e o afixo a. O triângulo é retângulo com catetos 1 e \sqrt{8}, tal que tg \alpha = \frac{1}{sqrt{8}}. Seja \theta o ângulo complementar. Então tg \theta = \sqrt{8}. Como \alpha + \theta = \frac{\pi}{2}, o ângulo que o afixo b formará com a horizontal será \theta, mas negativo pois tem de ser no quarto quadrante. Se b = x+yi, então \frac{y}{x} = \sqrt {8} \Rightarrow y = x\sqrt{8}. Como módulo é um: |b| = \sqrt { x^2 + y^2 } = 1 \Rightarrow x^2 + y^2 = 1 \Rightarrow x^2 + 8x^2 = 1 \Rightarrow x = \frac{1}{3} \Rightarrow y = \frac{\sqrt{8}}{3}.

Logo, o afixo é b = \frac{1 + i\sqrt{8}}{3}.